Download presentation
Presentation is loading. Please wait.
1
Questions, Answers, and Explanations
EENT Blueprint Questions, Answers, and Explanations
2
Question 1 1. Your patient is a 64 year old male was found to have an opacity on his lens and decreased visual acuity. Which of the following is the most likely diagnosis? A. Glaucoma B. Cataract C. Astigmatism D. Hyphema
3
Answer 1 1. Choice B is the correct answer. A cataract is a defect in the lens. Cataracts are the leading cause of blindness in the world. Only treatment is surgical correction. Astigmatism is a defect in the cornea. Glaucoma is characterized by elevated intraocular pressures. Hyphema is an accumulation of blood in the anterior chamber.
4
Question 2 2. Your patient is a 23 year old female who is a contact lens wearer. She presents with an injected right conjunctiva. She has no foreign body sensation or photophobia. There is no uptake of the stain and slit lamp exam is otherwise normal. Which eye drops are most ideal for this patient. A. Sulfacetamide B. Gentamicin C. Zithromax D. Ciloxan
5
Answer 2 2. Choice D is the correct answer. Contact lens wearers are prone to getting bacterial infections especially pseudomonas of the eye. Ciloxan has the best pseudomonas coverage. Zithromax and sulfacetamide does not cover pseudomonas.
6
Question 3 3. Your patient is a 34 year old male that has had maxillary sinus pressure and purulent drainage for 3 weeks. The patient is allergic to penicillin. He has had a history of anaphylaxis with penicillin. Which of the following is the most appropriate antibiotic choice? A. Augmentin B. Cephalexin C. Amoxicillin D. Bactrim
7
Answer 3 3. Choice D is the correct answer. The major organisms associated with bacterial acute sinusitis is streptococcus pneumoniae, H. influenzae, and M. Catarrhalis. Both Augmentin and Amoxicillin are both penicillins. Cephalexin is a cephalosporin that can have a 10 percent cross reactivity with penicillin allergic patients. With the patient's history of anaphylaxis with penicillin, the best choice is D bactrim.
8
Question 4 4. Your patient is a 64 year old male that presents with a right sided epistaxis that is on coumadin and ASA for Atrial Fib and CAD. His INR is You have successfully packed the right nostril with a rhino rocket. There is no further bleeding. Which of the following is the best management action? A. Stop the INR until his recheck in 2 days. B. Stop the ASA until his recheck in 2 days C. Place the patient on Amoxicillin 500 mg three times a day and have the patient follow up with ENT in 2 days. D. Have the patient pull out his own packing in two days.
9
Answer 4 4. Choice C is the correct answer. When hemostasis can be achieved and there is no indication for holding or reversing the patient's coumadin, especially with a therapeutic INR. ASA should not be stopped either. The effects ASA has on platelet aggregation actually last for 7 days, so it is actually pointless to hold the patient's ASA and risk sequela. The patient should not pull out his packing in two days. This needs to be done in a controlled environment by a skilled practitioner that can deal with any potential airway issues that may arise.
10
Question 5 5. Which of the following organisms is most likely to cause Parotitis? A. Staphylococcus Aureus B. Streptococcus Viridans C. Streptococcus Pneumoniae D. Haemophilus Influenzae
11
Answer 5 5. Choice A is the correct answer. Staphylococcus Aureus is most likely cause of parotitis when it is bacterial. It can also be caused by mixed aerobes and anaerobes of the mouth. Nafcillin and Metronidazole are recommended for Parotitis.
12
Question 6 6. Which of the following is the most concerning foreign bodies to encounter in a nose? A. Button Battery B. Corn C. Back of an Earring D. Candy
13
Answer 6 6. Choice A is the correct answer. Corn and candy are not dangerous but need to be removed and are not as time sensitive. A back of an earring and button battery are both solid objects and could be potentially aspirated but unlikely in a conscious, healthy patient. However, button batteries are more dangerous because electrolysis at the negative battery pole causes hydrogen ions that cause an alkaline environment and tissue necrosis.
14
Question 7 7. Your patient is a 34 year old female that presents with a fever 102 degrees F, exudative tonsillitis, runny nose, productive cough, and anterior cervical lymphadenopathy. Which of the following is the best management option? A. Start Bactrim empirically to cover for Group A Strep B. Obtain a Rapid Strep Test C. Start Amoxicillin empirically to cover for Group A Strep D. Wait on throat culture results in 2 days to determine treatment
15
Answer 7 7. Choice B is the correct answer. The Centor Criteria includes the following: exudative tonsillitis, fever, anterior cervical lymphadenopathy and absence of cough. According to the Centor Criteria, if the patient has 3 or more of the Centor Criteria, a rapid strep should be obtained. If there are two or less of the criteria, neither a rapid strep nor a throat culture are indicated. Bactrim does not do a good job against strep. Amoxicillin is indicated if the strep test or throat culture is positive for strep. Throat cultures will be ordered if the strep test is negative.
16
Question 8 8. All of the following may be exacerbating factors for the condition depicted below except? A. HPV Infection B. UV Light C. Contact Lens Wearers D. Abnormal HLA expression
17
Answer 8 8. Choice C is the correct answer. Contact lenses have not been identifying as a potential exacerbating factor for pterygium. Exacerbation factors of pterygium that have been defined include: HPV infection, UV light, abnormal HLA expression, abnormal conjunctival tumor suppressor gene, and the presence of angiogenesis related factors.
18
Question 9 9. Your patient is a 64 year old female that presents with gradual onset of peripheral bilateral vision loss over the last year. Which of the following is the most likely diagnosis is the most likely diagnosis? A. Strabismus B. Optic Neuritis C. Macular Degeneration D. Open Angle Glaucoma
19
Answer 9 9. Choice D is the correct answer. Open angle glaucoma is an optic neuropathy that has a progressive peripheral visual field loss and is eventually followed by central visual loss. Cataracts have central vision loss. Macular degeneration and Optic Neuritis both cause central vision loss. Strabismus is when the eyes do not line up correctly.
20
Question 10 10. Your patient is a 54 year old male that presents with a runny nose and cough. The patent is also complaining of vertigo, tinnitus, and right sided hearing loss. Which of the following is the most likely diagnosis? A. Labyrinthitis B. Acute Vestibular Neuritis C. Cerumen impaction D. Benign Postural Positional Vertigo
21
Answer 10 10. Choice A is the correct answer. Both vestibular neuritis and labyrinthitis have vertigo with usually a viral syndrome proceeding it. The key characteristic is there is no hearing loss associated with acute vestibular neuritis. Cerumen impaction will not cause tinnitus typically and usually does not have a viral syndrome associated with it. BPPV has recurrent episodes of vertigo associated with predictable head movements. There is usually no tinnitus associated with it.
22
Question 11 11. Which of the following have not been identified as a risk factor for barotrauma to the tympanic membrane? A. Diving B. Flying C. Allergic Rhinitis D. Blast Injuries
23
Answer 11 11. Choice C is the correct answer. Diving, flying, and blast injuries are identified as circumstances predisposing patients to barotrauma. Barotrauma is caused by pressure difference between the outside world and the inside of the middle ear. Allergic rhinitis can cause eustachian tube dysfunction but usually does not cause barotrauma. There usually is significant pressure difference between the middle ear and the outside world when the patient is exposed to pressure differences of diving, flying or blast injuries.
24
Question 12 12. Which of the following areas is the most common site for epistaxis to occur? A. Arterioles off the external carotid artery B. Arterioles off the internal carotid artery C. Stensen's Plexus D. Kiesselbach's Plexus
25
Answer 12 12. Choice D is the correct answer. Ninety percent of nose bleeds arising from Kiesselbach's Plexus. Stensen's Plexus does not exist, it is a duct that drains the parotid glands into the oral cavity. The tributaries of the internal carotid artery to supply blood to the posterior nasopharynx and can be the source of some arterial bleeds.
26
Question 13 13. All of the following are causes of otitis externa except? A. Trauma B. Infection C. Allergic Contact Dermatitis D. Cerumen Impaction
27
Answer 13 13. Choice D is the correct answer. Cerumen impaction in and of itself is not an etiologic factor of otitis externa. Aggressive cleaning abrading the external auditory canal can. Trauma, infection, and allergic contact dermatitis can cause otitis externa.
28
Question 14 14. Which of the following is the most common organism causing mastoiditis? A. H. Influenzae B. S. Aureus C. S. Pneumoniae D. Group B Hemolytic Strep
29
Answer 14 14. Choice C is the correct answer. Streptococcus pneumoniae is the most common organism that causes mastoiditis. Mastoiditis is the most common suppurative complication of otitis media. Pseudomonas is another consideration especially on recurrent infections with a ruptured tympanic membrane.
30
Question 15 15. Which of the following is not a potential sequelae of a blow out fracture? A. Edema or hemorrhage of the inferior rectus muscle B. Entrapment of the inferior rectus muscle C. Exophthalmos D. Sensory loss in the inferior orbital nerve distribution
31
Answer 15 15. Choice C is the correct answer. Enophthalmos not exophthalmos is a complication of a blowout fracture. The complications from a blowout fracture stem from entrapment of the inferior rectus muscle. Ischemia, edema, and hemorrhage of the inferior rectus muscle result from this. There also can be some sensory loss in the inferior orbital nerve distribution as a result of this.
32
Question 16 16. Your patient is a 23 year old female that presents with swelling and erythema to the upper eyelid on her right eye. Her extraocular movements are intact. There is no pain with extraocular movements. She also has a history of Rosacea. Which of the following is the most likely diagnosis? A. Hordeolum B. Blepharitis C. Chalazion D. Dacryocystitis
33
Answer 16 16. Choice B is the correct answer. Blepharitis is characterized by swelling of the eyelid. Blepharitis is best treated with warm compresses and topical antibiotics such as bacitracin, erythromycin, or azithromycin. Chalazion is an inflammatory lesion that develops with meibomian glands becomes obstructed. Dacryocystitis is caused by the nasolacrimal gland being blocked. Hordeolum (stye) is a painful red lump on the eyelid when the gland on the edge of the eyelid gets inflamed.
34
Question 17 17. Your patient is a 19 year old female that presents with generalized headaches and blurry vision out of both eyes for several months. Physical exam reveals swelling around both optic discs, and is otherwise normal. All of the following are potential etiologies of this likely disorder except: A. Brain Tumor B. Benign Intracranial Hypertension C. Obstructive hydrocephalus D. Parkinson's
35
Answer 17 17. Choice D is the correct answer. Parkinson's is characterized by a decreased production of Dopamine from the Substantia Nigra of the brain. Parkinson's does not lead to increased intracranial pressure. Papilledema can be caused by brain tumors, increased CSF production, decreased CSF reabsorption and obstructive hydrocephalus.
36
Question 18 18. Your patient is a 54 year old male that presents with headaches, decreased hearing out of right ear, right side facial paresthesias. The MRI of brain reveals a 2 x 3 cm right sided acoustic neuroma. Which of the following is not true regarding acoustic neuromas? A. Chemotherapy is the cornerstone of therapy B. Bilateral acoustic neuromas should increase a suspicion for neurofibromatosis C. Larger tumors may cause ataxia D. You can have decreased taste or taste disturbances with these
37
Answer 18 18. Choice A is the correct answer. Treatment management options for acoustic neuroma include surgery, radiation, and observation. Larger tumors can cause compression of adjacent structures which lead to facial and trigeminal nerve pain, and paresthesias. Because these nerves innervate the tongue they can affect taste.
38
Question 19 19. Your patient is a 45 year old male that presents with episodic vertigo, hearing loss, and tinnitus over the last year. His Brain MRI was unremarkable. He has had no preceding viral syndrome. Which of the following is the most likely diagnosis? A. Benign Postural Peripheral Vertigo B. Vestibular Neuritis C. Labyrinthitis D. Meniere's Disease
39
Answer 19 19. Choice D is the correct answer. Meniere's disease is characterized by episodic vertigo, hearing loss and tinnitus. Labyrinthitis has similar symptoms but do not last as long and has a preceding viral infection. Vestibular neuritis does not have any ear symptoms nor does Benign Postural Peripheral Vertigo.
40
Question 20 20. A 54 year old male presents with fever, trismus, dysphagia, and a sore throat. Which of the following is the most likely diagnosis? A. Parotitis B. Sialadenitis C. Peritonsillar Abscess D. GERD
41
Answer 20 20. Choice C is the correct answer. Peritonsillar abscess present with trismus, fever, sore throat and hot potato voice. Parotitis, GERD, and Sialadenitis do not have a fever. Sialadenitis and parotitis can have trismus. Peritonsillar abscess are typically caused by Group A Strep. They can compromise the upper airway. Empiric treatment is with Unasyn or Clindamycin. Ultimate treatment is incision and drainage.
42
Question 21 21. Which of the following is the most common cause for hearing loss? A. Acoustic Neuroma B. Presbycusis C. Cerumen impaction D. Vestibular Neuritis
43
Answer 21 21. Choice C is the correct answer. Both acoustic neuroma and presbycusis have hearing loss. Vestibular neuritis doses not. Cerumen impaction by far is the most common cause of hearing loss. Presbycusis is age related hearing loss.
44
Question 22 22. Your patient is a 45 year old male that presents with with right sided mastoiditis with a perforated right TM from an otitis media. This is his third episode of mastoiditis. Which of the following is the most appropriate choice of antibiotics? A. Unasyn B. Clindamycin C. Cipro D. Zithromax
45
Answer 22 22. Choice C is the correct answer. Pseudomonas should be suspected for recurrent mastoiditis or mastoiditis with otitis media with perforated tympanic membrane. Cipro is the antibiotic of choice with the best sensitivity against pseudomonas.
46
Question 23 23. Your patient is a 63 year old female that has a history of HTN and CAD. She has had a 3 month history of recurrent episodes of vertigo. These episodes tend to occur when she lays back in bed with her head not propped up. Her ENT exam is unremarkable. She has had an MRI of her brain that was negative as well as carotid dopplers. Which of the following is the most likely diagnosis? A. Meniere's Disease B. Benign Paroxysmal Positional Vertigo C. Vestibular Neuritis D. Cerebellar Infarct
47
Answer 23 23. Choice B is the correct answer. Benign Paroxysmal Positional Vertigo tends to have brief recurrent episodes of vertigo with predictable head movements or positions. There is no auditory symptoms. BPPV usually has a positive Hallpike Maneuver. Meniere's disease usually has hearing loss and tinnitis with vertigo. Cerebellar infarct is ruled out because of the negative MRI.
48
Question 24 24. Your patient is a 45 year old male that presents with anterior purulent drainage, facial pain, and decreased smell for 6 months. Which of the following is not recommended for management of this disorder? A. Surgery B. Topical Nasal Steroids C. Decongestants D. Doxycycline BID for 8 weeks
49
Answer 24 24. Choice D is the correct answer. Chronic sinusitis is an inflammatory condition that involves the paranasal sinuses and persists for greater than 12 weeks. The use on antibiotic monotherapy is limited treated has been directed towards controlling the inflammation. Decongestants, topical steroids, antihistamines, and leukotriene inhibitors are beneficial. Surgery is sometimes indicated to clear the sinus passages and get cultures.
50
Question 25 25. Your patient is a 72 year old female that presents with recurrent nose bleeds of right nostril and has right sided cervical lymphadenopathy. She is not on any anticoagulants. Her labs including INR, CBC and PTT are normal. Which of the following is the most likely diagnosis? A. Blood dyscrasia B. Thrombocytopenia C. AV malformation D. Malignancy
51
Answer 25 25. Choice D is the correct answer. The patient's labs are normal so blood dyscrasias and thrombocytopenia is not likely. While AV malformation is a plausible explanation, the fact there is some right sided lymphadenopathy with recurrent nosebleeds should rise the suspicion for malignancy.
52
Question 26 26. Which of the following disorders is characterized by a "thumb sign" on a soft tissue neck x ray? A. Laryngitis B. Croup C. Epiglottitis D. Foreign Body
53
Answer 26 26. Choice C is the correct answer. Epiglottitis is characterized by a thumb sign. Croup has a steeple sign. Laryngitis and foreign bodies do not have any characteristic signs on soft tissue neck x rays.
54
Question 27 27. Your patient is a 23 year old female that presents with white plaques on tongue. She has no history of chemotherapy, dentures, oral inhaled glucocorticoids, or radiation therapy. She is not a diabetic. Which of the following is the most likely etiology of her symptoms? A. HIV B. Smokeless tobacco C. Aphthous Ulcers D. HSV
55
Answer 27 27. Choice A is the correct answer. Whenever there is unexplained thrush is a health individual, HIV should be considered as an etiology. Smokeless tobacco is a risk factor for leukoplakia. Aphthous ulcers produce ulcers not white plaques. HSV presents as vesicles.
56
Question 28 28. Your patient is a 32 year old female that presents with a nodular rubbery lesion to her upper eyelid. It is not painful. It has been there for 6 days. Which of the following is the most likely diagnosis? A. Chalazion B. Hordeolum C. Dacryocystitis D. Foreign Body
57
Answer 28 28. Choice A is the correct answer. A chalazion is an inflammatory lesion that developed when the meibomian tear gland becomes obstructed. They are usually not painful. Treatment is warm compresses or surgical correction. Hordeolum and stye are painful. Dacryocystitis is caused by the nasolacrimal duct being blocked.
58
Question 29 29. Your patient is a 21 year old male that presents with redness to right eye, photophobia, and tearing. Your patient intraocular pressure is 18. Slit lamp exam and staining are normal. The patient is not a contact lens wearer. When the patient was anesthetized with alacaine and still has pain. Which of the following is the most likely diagnosis? A. Conjunctivitis B. Corneal Abrasion C. Iritis D. Glaucoma
59
Answer 29 29. Choice C is the correct answer. The patient has normal intraocular pressures so glaucoma is not likely. Conjunctivitis and corneal abrasion are not likely. The slit lamp and staining are normal so this rules that out. Iritis presents as a red eye, photophobia, and tearing. Iritis is often not improved when the eyes are anesthetized with alcaine.
60
Question 30 30. What is the treatment for eye condition depicted below? A. Mydriasis B. Prednisolone eye drops C. Surgery D. Xalatan Eye Drops
61
Answer 30 30. Choice C is the correct answer. Most hyphema need to be treated surgical drainage. Mydriatics can provide some symptomatic relief. Prednisolone eye drops are for inflammatory conditions of the eye. Xalatan eye drops are for glaucoma.
62
Question 31 31. Your patient is a 34 year old male that presents with right eye pain, photophobia, and a scratch of the cornea with staining. There was no hyphema. Which of the following is not part of treatment of this condition? A. Gentamicin eye drops B. Oral analgesics C. Sunglasses D. Patching
63
Answer 31 31. Choice D is the correct answer. Wearing sunglasses and oral analgesics help with the pain associated with corneal abrasions. Gentamicin eye drops or any eye drops help with preventing infection and lubricate the eye. Contact lens wearers need to be on eyedrops such as ciloxan for anti pseudomonas activity. Patching is never recommended in treatment of corneal abrasion.
64
Question 32 32. Your patient is a 68 year old female that presents with several months of progressively worsening central vision loss. Physical exam reveals Drusen Spots of the eye. Which of the following is the most likely diagnosis? A. Macular Degeneration B. Open Angle Glaucoma C. Optic Neuritis D. Cataracts
65
Answer 32 32. Choice A is the correct answer. Macular degeneration causes central vision loss and can have Drusen Spots. Open angle glaucoma typically causes peripheral vision loss. Optic neuritis is more of an acute onset. Cataracts do not have a Drusen Spots.
66
Question 33 33. Your patient is a 34 year old male presents with a erythema and swelling to the upper right medial nasolacrimal duct. He has no fever. His extraocular movements are intact. He has no ectropion. Which of the following is the most likely diagnosis? A. Periorbital cellulitis B. Dacryocystitis C. Orbital Cellulitis D. Hordeolum
67
Answer 33 33. Choice B is the correct answer. Dacryocystitis is caused by the nasolacrimal duct being blocked. Most common organism is alpha hemolytic strep, staph epidermidis, and staph aureus. Empiric therapy with clindamycin and doxycycline or bactrim is helpful. This can lead to periorbital cellulitis. Orbital cellulitis is an infection that involves the fat and muscles around the eye. Periorbital cellulitis is an infection of the anterior portion of the eyelid.
68
Question 34 34. Your patient is a 3 year old male that presents with a bulging right tympanic membrane and a fever of He has just finished a course of amoxicillin 14 days ago. Which of the following is the most appropriate treatment? A. High dose amoxicillin B. Augmentin C. Keflex D. Cipro
69
Answer 34 34. Choice B is the correct answer. This patient is at risk for beta lactam resistance therefore keflex and amoxicillin should not be used. The patient is only 3, and cipro should be avoided in this age group.
70
Question 35 35. Which of the following is the most appropriate management for a auricular hematoma? A. Observation B. Incision and drainage C. Ice are 20 minutes 4 times a day D. Start cephalexin
71
Answer 35 35. Choice B is the correct answer. Auricular hematomas should be drained as soon as possible. Delay in treatment can lead to a cauliflower ear that is permanent deformity of the fibrocartilage of the ear that is an overgrowth. Observation, antibiotics, and icing would alone are not effective.
72
Question 36 36. Which of the following is not an etiology of tinnitus?
A. ASA B. Congenital AV Malformations C. Cerumen Impaction D. Vascular pathology
73
Answer 36 36. Choice C is the correct answer. Cerumen impaction is not an etiology of tinnitus. ASA, vascular problems, congenital AV malformations, hearing loss, eustachian tube dysfunction, and ototoxic medications are etiologies for tinnitus.
74
Question 37 37. Your patient is a 18 year old female that presents with right side purulent otorrhea. Visualized tympanic membrane on the right side reveals a small perforation. Which of the following is the most appropriate treatment regimen? A. Amoxicillin B. Floxin Otic Drops C. Amoxicillin and Floxin Otic Drops D. Zithromax
75
Answer 37 37. Choice C is the correct answer. Perforated tympanic membrane caused by infection needs to be treated with oral antibiotics as well as topical antibiotic ear drops. When there is a perforation the ear canal is contaminated with the purulent drainage
76
Question 38 38. Which of the following conditions are not associated with allergic rhinitis? A. Atopic Dermatitis B. Urticaria C. Sinusitis D. Asthma
77
Answer 38 38. Choice B is the correct answer. Atopic Dermatitis, sinusitis, and asthma are common diseases seen with allergic rhinitis. In additional allergic conjunctivitis is commonly seen with allergic rhinitis. Urticaria does not have an association with allergic rhinitis.
78
Question 39 39. Which of the following bacteria is not a possible cause of bacterial tonsillitis? A. Group A Streptococcus B. Neisseria Gonorrhea C. Mycoplasma D. Proteus
79
Answer 39 39. Choice D is the correct answer. All of the following are possible causes of bacterial tonsillitis: Group A Streptococcus, Neisseria Gonorrhoeae, Mycoplasma, Chlamydia, Tularemia, and Group C and Group G Strep. Group C and Group G Strep do not cause rheumatic fever. Viral etiologies include CMV, HSV, Mononucleosis, and Influenza.
80
Question 40 40. Your patient is a 45 year old male that is not immunized. He presents with a fever of 102, dysphagia, drooling, and shortness of breath. Lungs are clear. O2 Sat 100%. Pulse RR-24. Soft tissue neck reveals a thumb sign. All of the following are acceptable management plans except: A. Give a Racemic Epinephrine aerosol treatment and discharge the patient home on steroids B. Begin Vancomycin and Ceftriaxone C. Intubate the patient in the operation room D. Consult anesthesia or ENT
81
Answer 40 40. Choice A is the correct answer. Racemic epinephrine can help momentarily, but ultimately the patient needs intubated and admitted to the hospital. Airway protection is the mainstay of treatment. The patient usually needs intubated for 2-3 days prior to weaning attempts. The role of steroids is controversial. Vancomycin helps with anti-staph coverage and ceftriaxone covers the most common organism haemophilus influenzae type B. Anesthesia and/or ENT should be consulted for airway management.
82
Question 41 41. Which of the following is not true about peritonsillar abscess? A. Group A Streptococcus is the most common organism B. MRI of the soft tissue neck is the imaging study of choice for confirmation C. Appropriate empiric antibiotic therapy includes clindamycin or unasyn D. Vancomycin is an appropriate add on antibiotic if there are airway issues
83
Answer 41 41. Choice B is the correct answer. CT scan of soft tissue of the neck with IV contrast is the imaging study of choice. Group A Streptococcus is the most common organism. There are also so cases of staph. Appropriate empiric antibiotic therapy includes clindamycin and unasyn. Vancomycin is an appropriate add on antibiotic when there are airway issues.
84
Question 42 42. All of the following are potential etiologies for parotitis except? A. Stensen's Duct obstructed with a stone B. Retrograde salivary stasis and seeding in Stensen's Duct C. Staphylococcus aureus D. Streptococcus Viridans
85
Answer 42 42. Choice D is the correct answer. Potential etiologies of parotitis include: anything that obstructs Stensen's Duct (Stone or Tumor), poor oral hygiene, dehydration, retrograde salivary stasis and seeding of Stensen's Duct with oral flora, viruses, or staphylococcus aureus.
86
Question 43 43. Your patient is 54 year old immunocompetent male that presents with right sided parotitis, trismus, and a fever of Which of the following antibiotic regimens are most appropriate? A. Nafcillin and Metronidazole B. Nafcillin C. Metronidazole D. Azithromycin
87
Answer 43 43. Choice A is the correct answer. Empiric therapy for parotitis involves nafcillin and metronidazole. Vancomycin and cefepime or imipenem is recommended for immunocompromised patients. Azithromycin does not cover staph.
88
Question 44 44. Your patient is a 54 year old male that presents with erythema and redness to the right submandibular area. There is no pain to palpation over the floor of the mouth. Physical exam of the mouth reveals pus covering from the salivary duct. There is no dental pain or trismus. Which of the following is not true regarding the patient's condition? A. Usually resolves in 7-10 days with antibiotics B. Typically responds to keflex or dicloxacillin C. If the diagnosis is in question an ultrasound of the neck in the best imaging modality D. Can be caused by salivary duct stone
89
Answer 44 44. Choice C is the correct answer. Imaging modality of choice for sialadenitis is a CT scan of soft tissue neck with IV contrast. The usual organism when involved is staph. This is generally sensitive to dicloxacillin or keflex. Usually resolves itself within 7-10 days with antibiotic treatment.
90
Question 45 45. Your patient is a 65 year old male that presents with Fordyce spots noted in his oral cavity. What is the the likely etiology of these Fordyce spots? A. Mucocele B. Melanoma C. Amalgam Tattoo D. Benign Sebaceous Cyst Tumor
91
Answer 45 45. Choice D is the correct answer. Fordyce spots are associated with benign sebaceous cyst tumors. Melanomas are oral pigmented lesions that have irregular borders and symmetry. A mucocele is a fluid filled cavity with mucus glands lining the epithelium. Amalgam tattoos are black macules seen near the gingiva of dental fillings.
92
Question 46 46. Which of the following is not true regarding entropion? A. It is a turning in of the edge of an eyelid B. Artificial tears help with dryness C. Usually affects the upper eyelid D. Trachoma infection can cause lower lid infection
93
Answer 46 46. Choice C is the correct answer. Entropion usually affects the lower eyelid. Trachoma infection can cause but rarely seen in the US. Artificial tears help with dryness and surgical treatment is needed to correct.
94
Question 47 47. Your patient is a 46 year old male that presents with a piece of metal on cornea with a rust ring that is approximately twelve hours old. Which of the following are acceptable methods to attempt removal? A. 25 gauge needle B. Cotton Swab C. Magnet D. Ophtho Burr
95
Answer 47 47. Choice C is the correct answer. Magnets are not safe for removal metal foreign bodies from the eye. A cotton swab, 25 gauge needle, or an ophtho burr are acceptable methods for removal.
96
Question 48 48. Your patient is 64 year old female that presents with sudden onset of right eye pain, redness, and photophobia with tearing. Slit lamp exam reveal some conjunctival erythema and a shallow anterior chamber. There is no ciliary flair. The intraocular pressure is 44 in the right eye. All of the following are acceptable management options except? A. Referral to Ophthalmology B. Begin Xaltan Drops C. Instill Cycloplegic Drops D. Start Norco as needed
97
Answer 48 48. Choice C is the correct answer. This patient has acute closure glaucoma. Instillation of a cycloplegic can increase the pressure, make the patient worse. Xalatan drops help lower the pressure. Analgesics PRN are appropriate. This patient should be referred to ophthalmology.
98
Question 49 49. Your patient is a 45 year old female that presents with a sudden ones of visual loss in the left eye. Fundoscopic exam reveals hyperemia and swelling and blurring of the optic disc. There is no history of headache. The patients other eye is normal. The patient describes a scotoma in the central vision of his affected eye. Based upon this data, which of the following is the most likely diagnosis? A. Pseudotumor cerebri B. Ophthalmic migraine C. Retinal detachment D. Optic Neuritis
99
Answer 49 49. Choice D is the correct answer. Pseudotumor cerebri usually affects both eyes and is accompanied with a headache. Ophthalmic migraine usually has headaches with it and will not have any optic disc changes on exam. Retinal detachment does not have these visual field changes and will not have these optic disc changes on physical exam.
100
Question 50 50. Your patient is a 69 year old female that presents with a sudden onset of floaters in the right eye, flashes of light, and floaters resemble a cobweb. There is no pain. Which of the following is the most likely diagnosis? A. Optic Neuritis B. Retinal Detachment C. Pseudotumor cerebri D. Macular Degeneration
101
Answer 50 50. Choice C is the correct answer. Optic neuritis does not have this presentation. Pseudotumor cerebri typically affects both eyes and will have a headache. Macular degeneration is more of a slower onset and does not have these visual field changes typically.
102
Question 51 51. On physical examination with otoscopy, where is the MOST common location of a cholesteatoma? A. Umbo B. Pars tensa C. Pars flaccida D. External Canal E. Cone of Light Reflex
103
Answer 51 51. Choice C is the correct answer. The Pars Flaccida is the most common location of the cholesteatoma.
104
Question 52 You are evaluating a 19 year-old male who complains of an itchy nose and sneezing that started when he moved to the area 4 weeks ago. He also has a scratchy throat. He denies fever, chills, shortness of breath, malaise, and rash. On physical examination you note purplish, swollen turbinates and clear rhinorrhea. Which one of the following is the mainstay of treatment for this condition? A. Nasal saline irrigation B. Oral decongestant C. Intranasal steroid spray D. Oral antibiotic E. Oral non-sedating antihistamine
105
Answer 52 52. Choice C is the correct answer. Intranasal steroid spray is the mainstay for allergic rhinitis.
106
Question 53 53. Which one of the following is a useful tool to differentiate conductive hearing loss from sensorineural hearing? A. Whisper test B. Otoscope C. Tuning fork D. Tympanogram E. Calorimeter
107
Answer 53 53. Choice C is the correct answer. A tuning fork can help differentiate between conductive and sensorineural hearing.
108
Question 54 54. Pain and erythema in the postauricular area that is associated with a spiking fever is MOST LIKELY due to which one of the following? A. Pneumonia B. Mastoiditis C. Rheumatic fever D. Serous otitis media E. Sialadenitis
109
Answer 54 54. Choice B is the correct answer. This patient has mastoiditis.
110
Question 55 55. A patient presents with otalgia, pruritus and a purulent discharge from the left ear. Physical examination reveals an afebrile child with erythema and edema of the ear canal, pain with manipulation of the auricle, and a normal tympanic membrane. Which one of the following is the MOST LIKELY diagnosis? A. Acute otitis media B. Serous otitis media C. Eustachian tube dysfunction D. Otitis externa E. Malignant otitis
111
Answer 55 55. Choice D is the correct answer. This patient has otitis externa.
112
Question 56 56. You are evaluating a 37 year-old female who complains that it is getting harder to understand what people are saying when they are positioned to her left. She also complains of mild continuous vertigo. Which one of the following is the BEST study for evaluation of this condition? A. Pure tone audiography B. Computed tomography scanning C. Magnetic resonance imaging D. Auditory brainstem response E. Trans-tympanic vestibulography
113
Answer 56 56. Choice C is the correct answer. This patient needs an MRI to look for any signs of infarct or space occupying lesion. The patient’s symptoms seem peripheral in nature.
114
Question 57 57. You are evaluating a 54 year-old woman who has had episodic vertigo, tinnitus, hearing loss and aural pressure. She has a negative Dix-Hallpike maneuver and there are no carotid bruits. What is the intended goal of treatment? A. Increase urination B. Decrease nystagmus C. Lower endolymphatic pressure D. Adequate control nausea and vomiting E. Lower serum calcium
115
Answer 57 57. Choice C is the correct answer. The goal of therapy is to lower the endolymphatic pressure.
116
Question 58 58. You are evaluating a 21 year-old male who complains of severe sore throat and fever. On examination you note trismus, deviation of the uvula base and peritonsillar fold and a "hot potato" voice. Which one of the following is the MOST LIKELY diagnosis? A. Strep pharyngitis B. Viral pharyngitis C. Ludwig’s Angina D. Peritonsillar abscess E. Retropharyngeal abscess
117
Answer 58 58. Choice D is the correct answer. This patient has a peritonsillar abscess.
118
Question 59 59. A 35 year-old female presents to your family practice clinic complaining of hearing loss, tinnitus, and severe vertigo that began suddenly. The patient reports a viral upper respiratory illness that started about one week ago but is getting better. What is the MOST LIKELY diagnosis? A. Meniere's syndrome B. Otitis media C. Acute sinusitis D. Labyrinthitis E. Eustachian tube disfunction
119
Answer 59 59. Choice D is the correct answer. This patient has labyrinthitis. This presents with a viral syndrome preceding hearing loss, vertigo, and tinnitus that begins quickly.
120
Question 60 60. You are evaluating a 15 year-old female who complains of pulsatile tinnitus in the left ear that is getting progressively worse over the past month. On examination you note reddish-blue pulsing left middle ear mass and slight left sided facial droop What test is ordered to establish the diagnosis? A. MRI B. CT window C. MR angiography D. Caloric testing E. Electronystagmography
121
Answer 60 60. Choice C is the correct answer. Glomus Tumors are best diagnosed with a MRA of the area.
122
Question 61 61. You are called to the Emergency Department to evaluate a 20 year-old male who presented complaining of pain and swelling under his “chin”. VS: BP 130/76, P 104, R 18, T 101 F. On examination you note the patient has poor dentition and a swollen, tender, firm, mass in the submental - submaxillary region. The tongue is pushed upward and the airway is compromised. Which one of the following is the MOST LIKELY diagnosis? A. Vincent’s angina B. Ludwig’s angina C. Lingual abscess D. Peritonsillar abscess E. Retropharyngeal abscess
123
Answer 61 61. Choice B is the correct answer. Ludwig’s angina is fever, submandibular swelling and is an abscess on the floor of the oral cavity.
124
Question 62 62. Which one of the following is the BEST initial treatment for anterior epistaxis? A. Use Afrin in the nostril B. Electrocautery C. Petroleum jelly nasal packing D. Apply silver nitrate to the nasal septum E. Firmly pinch the nostrils shut for 5 to 10 minutes
125
Answer 62 62. Choice E is the correct answer. The best way to control the bleeding is direct pressure to the nostrils. Ninety percent of nosebleeds arise from Kiesselbach’s plexus.
126
Question 63 63. You are evaluating a patient who complains of a painful area at the tip of the nose just inside the left nostril. On examination you note a furuncle in the left nasal vestibule. Adequate treatment is important to prevent which of the following? A. Acute sinusitis B. Retrograde infection C. Diffuse cellulitis D. Rhinocerebral mucormycosis E. Olfactory dysfunction
127
Answer 63 63. Choice B is the correct answer. Treatment is necessary to prevent retrograde infection.
128
Question 64 64. Individuals with eustachian tube dysfunction should receive counseling from their provider on the risk of barotrauma during all the following conditions EXCEPT ? A. Swimming B. Platform diving C. Airplane descending D. Underwater scuba diving E. Airplane ascending
129
Answer 64 64. Choice A is the correct answer. Patients with eustachian tube dysfunction are not at risk for barotrauma with regular swimming.
130
Question 65 65. All of the following is a relative or absolute indication for tonsillectomy EXCEPT? A. Snoring B. Recurrent strep tonsillitis C. Chronic tonsillitis D. Recurrent peritonsillar abscess E. Airway obstruction causing sleep apnea
131
Answer 65 65. Choice A is the correct answer. Snoring is not an indication for tonsillectomy.
132
Question 66 66. You are evaluating a 48 year-old male who complains of a colored “patch” of skin inside his mouth. On examination of the buccal mucosa you note an erythematous patch that cannot be scraped off with a tongue blade. What is the chance (percentage) that this lesion is dysplastic or carcinoma? A. 10% B. 25% C. 70% D. 80% E. 90%
133
Answer 66 66. Choice E is the correct answer. Ninety percent of these type of lesions will be dysplastic or carcinoma.
134
Question 67 67. A 23 year-old college student presents to your clinic complaining of a sore throat, anorexia, sweats, chills and extreme fatigue x 10 days. Physical examination reveals an acutely ill male with oral temperature of F, heavy white tonsillar exudates, swollen tonsils, diffuse cervical adenopathy, and palpable tender splenomegaly. Which one of the following tests would confirm the MOST LIKELY diagnosis? A. Rapid Strep B. Throat culture C. 5% atypical lymphocytosis D. HIV ELISA E. Heterophil agglutination test
135
Answer 67 67. Choice E is the correct answer. A monospot test is also known as a Heterophil Agglutination Test.
136
Question 68 68. After evaluating a 20 year old patient, you are concerned with patient compliance. The patient has no known drug allergies, which one of the following is the drug of choice in the treatment of acute streptococcal pharyngitis? A. Benzathine penicillin 0.6 million units intramuscular injection B. Benzathine penicillin 1.2 million units intramuscular injection C. Penicillin V 500mg po every 12 hours for 10 days D. Erythromycin 250 mg po every 6 hours for 10 days E. Levofloxacin 750 mg po every 24 hours for 7 days
137
Answer 68 68. Choice B is the correct answer. Benzathine penicillin 1.2 million units times one dose IM will treat strep pharyngitis. It is especially good when there may be compliance issues.
138
Question 69 69. Which one of the following characteristics is closely associated with croup? A. Inspiratory-expiratory wheezes B. Purulent sputum and halitosis C. Barking cough with an ending 'whoop' D. Inspiratory stridor with a barking cough E. Gray membranous tonsillar exudate
139
Answer 69 69. Choice D is the correct answer. Inspiratory stridor with a barking cough is a characteristic closely associated with croup.
140
Question 70 70. A 53 year-old male presents to your clinic complaining of 3 months of hoarseness. He has a history of seasonal allergic rhinitis and a 2 pack a day smoking habit. Which one of the following is the MOST appropriate management of this patient? A. Encourage patient to stop smoking B. Initiate nasal corticosteroid treatment C. Initiate antibiotic treatment of laryngitis D. ENT evaluation by Indirect laryngoscopy E. Reassure patient hoarseness in secondary to allergies
141
Answer 70 70. Choice D is the correct answer. This patient needs a laryngoscopy by ENT to look for potential malignancy of the larynx.
142
Question 71 71. Which one of the following pathogens is MOST LIKELY to cause an external otitis? A. Klebsiella pneumoniae B. Escherichia coli C. Haemophilus influenzae D. Streptococcus pneumoniae E. Pseudomonas aeruginosa
143
Answer 71 71. Choice E is the correct answer. Pseudomonas is one of the most common causes of otitis externa.
144
Question 72 72. In the evaluation of a patient for suspected sinusitis, which one of the following is the MOST SENSITIVE for inflammatory changes? A. Antral window biopsy B. Sinus Culture C. Waters view plain film x ray D. Coronal “limited” CT scan E. Lateral xray of the sinuses
145
Answer 72 72. Choice D is the correct answer. Limited CT scan of sinuses coronal view is the most sensitive tests for inflammatory changes of the sinuses.
146
Question 73 73. You are evaluating a 15 year-old boy who has had a sore throat, fatigue and fever for the last 7 days. On examination you note pharyngeal exudates, diffuse cervical lymphadenopathy, and splenomegaly. Which one of the following is the MOST appropriate management of this condition? A. Interferon SQ B. Prednisone dose pack C. Nystatin swish and swallow D. Pen V 250mg po every 6 hours E. Supportive care with limited activity
147
Answer 73 73. Choice E is the correct answer. This patient clinically has mononucleosis. The treatment of this is supportive care and limitation of activity especially contact sports.
148
Question 74 74. Which one of the following is the MOST common a cause of conductive hearing loss? A. Cerumen impaction B. Middle ear effusion C. Otosclerosis D. Acoustic Neuroma E. Ossicular disruption
149
Answer 74 74. Choice A is the correct answer. Cerumen impaction is the most common cause of a conductive hearing loss.
150
Question 75 75. You are evaluating a 36 year-old woman who is HIV positive. She is complaining of severe ear pain and vertigo. An examination you note small grouped vesicles on an erythematous base at the right EAC and a right sided facial droop. Which one of the following is the MOST LIKELY diagnosis? A. Otitis externa B. Auricular cellulitis C. Ramsay-Hunt syndrome D. Referred otalgia E. Malignant external otitis
151
Answer 75 75. Choice C is the correct answer. Ramsay Hunt Syndrome is a herpetic infection of cranial nerve VII and CN VIII, IX, V and VI may also be involved.
152
Question 76 76. A patient presents to your clinic with acute parotid gland swelling and pain. The pain and swelling increase with meals. On examination you note tenderness and erythema at the opening of Stensen's duct. The patient reports having massaged pus from the duct. Which one of the following is the MOST LIKELY diagnosis? A. Lymphadenopathy B. Sjogren's syndrome C. Parotid gland tumor D. Sialolithiasis E. Sialadenitis
153
Answer 76 76. Choice E is the correct answer. Erythema at the opening of Stensen’s duct usually comes from a stone in the salivary duct. Sialadenitis is the correct answer.
154
Question 77 77. You are evaluating a 26 year-old who complains of a painful area inside her mouth. She is a 2 pack per day smoker. She is sexually active with more than one partner. Vital signs are BP 136/72, P 88, R 18, T 98.7F. On exam your notice the right buccal mucosa is reddened and has creamy white patches that you can scrape off with a tongue blade. This presentation may be the initial manifestation of which one of the following? A. Mucormycosis B. HIV infection C. EBV positive lymphoma D. Oral leukoplakia E. Ulcerative stomatitis
155
Answer 77 77. Choice B is the correct answer. This patient has oral candidiasis and likely has immunosuppression. Given her multiple partners this makes HIV a likely infection.
156
Question 78 78. Which one of the following is the MOST cost effective medication for the treatment of uncomplicated acute sinusitis for a penicillin allergy? A. Amoxicillin B. Azithromycin C. Trimethoprim-sulfamethoxazole D. Ciprofloxacin E. Clindamycin
157
Answer 78 78. Choice C is the correct answer. Bactrim is the most cost effective medication for the treatment of acute sinusitis with a penicillin allergy.
158
Question 79 79. You are evaluating a 25 year-old male who complains of a painful lesion just inside his mouth that he first noticed yesterday. On examination you note a 2 mm round ulceration with a yellow-gray center surrounded by a red halo found on the labial mucosa of the oral cavity. When counseling this patient, which one of the following is the MOST appropriate statement? A. Caused by a co-infection of spirochetes and fusiform bacilli B. The only effective treatment is a 10 day course of oral antibiotics C. Healing will be complete in 1 to 3 weeks D. The painful stage of this condition only last for two days E. This condition will not recur
159
Answer 79 79. Choice C is the correct answer. This patient has an Aphthous Ulcer. Aphthous ulcers heal in roughly 1-3 weeks.
160
Question 80 80. You are evaluating a 63 year-old female in the Emergency Department for a nose bleed that started about 15 minutes ago. It is early winter. VS: BP 154/92; P 88; R 18; T Which one of the following is MOST LIKELY associated with this epistaxis? A. Dry environment B. Poorly controlled HTN C. Nasal foreign body D. Forceful nose blowing E. Osler-Weber-Rendu syndrome
161
Answer 80 80. Choice A is the correct answer. Nosebleeds are common in dry environments because the tissue becomes friable. Also trauma to the nose is a common cause from nose picking.
162
Question 81 81. You are evaluating a 45 year-old male who complains of hearing loss in his right ear. The hearing loss has occurred slowly over the last 2-3 years. The patient also complains of ringing in the same ear. During physical examination you notice the Weber test lateralizes to the left ear and the Rinne test shows AC>BC in both ears. Which one of the following is the MOST likely diagnosis? A. Sensory hearing loss in the right ear B. Conductive hearing loss in the right ear C. Mixed hearing loss in the left ear D. Sensory hearing loss in the left ear E. Conductive hearing loss in the left ear
163
Answer 81 81. Choice A is the correct answer. This patient has sensory hearing loss in the right ear.
164
Question 82 82. A 43-year-old tri-athlete has severe ear pain. On examination the right auditory canal is very edematous, wet and erythematous with abundant debris. After careful irrigation, which of the following is the BEST treatment for this condition? A. Immediate referral to ENT specialist B. Insertion of wick with Cortisporin drops C. Pain medication and warm packs D. Instill antibiotic ointment E. Oral steroids
165
Answer 82 82. Choice B is the correct answer. This patient should have a wick inserted with cortisporin ear drops. If just cortisporin drops were inserted, it would not be effective because the canal is too swollen.
166
Question 83 83. You are evaluating a 36 year-old man who noted this morning while shaving that his face on the left was drooping. He is also having difficulty closing his left eye. Examination reveals a unilateral facial paralysis with no masses or lesions. Which one of the following pathogens is the MOST likely cause of these symptoms? A. Rhinovirus B. Mycoplasma C. Herpes virus D. Staphylococcus aureus E. Streptococcus pneumoniae
167
Answer 83 83. Choice C is the correct answer. Bell’s palsy has been associated with herpes virus.
168
Question 84 84. A mother brings her 2-year-old son into the ER. He presently has a barking cough and has had some URI symptoms for the past week. The child MOST LIKELY has: A. Croup B. Acute epiglottitis C. Foreign body obstruction D. Subglottic irritation E. Pneumonia
169
Answer 84 84. Choice A is the correct answer. This patient likely has croup.
170
Question 85 85. You are evaluating a 14-year-old girl who has had 24 hours of fever, sore throat, headache, and dysphagia. She has no history of previous URI symptoms. Examination reveals patchy tonsillar exudates, petechiae of the soft palate, and tender anterior cervical adenopathy. Which one of the following is the MOST LIKELY etiology of her condition? A. Viral upper respiratory pathogens B. Group A, ß-streptococcal infection C. Mycoplasmal pharyngitis D. Gonococcal pharyngitis E. Chlamydial pneumonia
171
Answer 85 85. Choice B is the correct answer. This patient has a Group A Beta Streptococcal infection.
172
Question 86 86. You are evaluating a 22 year-old female who complains of right sided facial pain and congestion for the last 8 days. She has had a fever of 100 to 101 and thick greenish, blood tinged rhinorrhea from the right nostril. On examination you note marked tenderness over the right maxillary sinus and mucopurulent rhinorrhea on the right. Which one of the following would be the MOST useful in the diagnosis of this condition? A. MRI with gadolinium B. Sinus Xrays C. CBC with differential D. Culture of rhinorrhea E. None, diagnosis can be made on clinical grounds
173
Answer 86 86. Choice E is the correct answer. This diagnosis can be made clinically.
174
Question 87 87. Which one of the following is MOST closely associated with the development of squamous cell carcinoma of the floor of the mouth? A. Recurrent submandibular sialadenitis B. History of neck irradiation C. Tobacco or alcohol history D. History of Papillomavirus E. Oral lichen planus
175
Answer 87 87. Choice C is the correct answer. Tobacco and alcohol history are closely associated with development of squamous cell carcinoma of the floor of the mouth.
176
Question 88 88. You are evaluating a 21 year-old male who complains of a tender mass on the left side of his neck. He first noticed it five days ago after “having a cold and sore throat”. The mass has gotten larger over the last few days. The patient denies any additional symptoms. On exam you note a slightly tender, ½ cm, mobile mass at the anterior aspect of the sternocleidomastoid muscle. The mass does not move when you have the patient swallow. Which one of the following is the MOST LIKELY diagnosis? A. Reactive lymphadenitis B. Thyroglossal duct cysts C. HIV D. Branchial cleft cyst E. Accessory thyroid tissue
177
Answer 88 88. Choice A is the correct answer. This patient has reactive lymphadenitis.
178
Question 89 89. You are evaluating a 32 year-old female who complains of pressure, popping and crackling in her ears, with decreased hearing for the past few days. She tells you she had a “cold” about a week ago but it got better. She states that swallowing and yawning cause the popping/crackling. On examination you note bilateral TM retraction. Which one of the following is the MOST LIKELY diagnosis? A. Acute otitis media B. Meniere’s disease C. Otosclerosis D. Serous otitis media E. Eustachian tube dysfunction
179
Answer 89 89. Choice E is the correct answer. This patient has eustachian tube dysfunction. This patient should be treated with nasal steroids.
180
Question 90 90. A 26 year-old woman is complaining of bilateral hearing loss. She is worried because she has a strong family history of early hearing loss. On examination you note a bilateral conductive hearing loss with normal TMs. Which one of the following is the MOST LIKELY diagnosis? A. Serous otitis media B. Perilymphatic fistula C. Meniere’s disease D. Acoustic neuroma E. Otosclerosis
181
Answer 90 90. Choice E is the correct answer. Otosclerosis would be the most likely cause of her conductive hearing loss.
182
Question 91 91. Which of the following medications should be limited to three days of regular use to avoid rebound congestion? A. Oxymetazoline (Afrin) B. Pheniramine (Triaminic) C. Guaifenesin (Robitussin) D. Chlorpheniramine (Dristan) E. Phenyltoloxamine (Comhist)
183
Answer 91 91. Choice A is the correct answer. Afrin should be limited to three days of regular use to avoid rebound nasal congestion.
184
Question 92 92. Which one of the following allergens is MOST LIKELY to cause a perennial allergic rhinitis? A. Dust mites B. Trees C. Grasses D. Mold E. Ragweed
185
Answer 92 92. Choice A is the correct answer. Dust mites are the most likely cause of perennial allergic rhinitis.
186
Question 93 93. You are evaluating a 36 year-old woman with a history of Meniere’s syndrome in the emergency department for acute onset of severe vertigo. She is vomiting and a little disoriented. Which of the following medications will offer the MOST rapid relief of her symptoms? A. ACE inhibitor B. Beta blockade C. IV Diazepam D. Ondansetron E. Nitroglycerin
187
Answer 93 93. Choice C is the correct answer. Valium intravenously would be the most rapid relief of her symptoms in this situation.
188
Question 94 94. Which one of the following tests allow for the separate stimulation of the vestibular system of each ear? A. Rinne B. Weber C. Caloric testing D. Rotary chair testing E. Brainstem-evoked response
189
Answer 94 94. Choice C is the correct answer. Caloric testing allows for separate stimulation of the vestibular system in each ear.
190
Question 95 95. Which one of the following is the MOST COMMON location for salivary calculi formation? A. Stensen’s duct B. Sublingual gland C. Submental gland D. Parotid gland E. Wharton’s duct
191
Answer 95 95. Choice E is the correct answer. Wharton’s duct is the most common location for salivary stone formation.
192
Question 96 96. Which one of the following symptoms is the common denominator among the various obstructive sleep syndromes? A. Regurgitation B. Chest pain C. Arrhythmia D. Snoring E. Obesity
193
Answer 96 96. Choice D is the correct answer. Snoring is one of the most common denominators in various obstructive sleep apnea syndromes.
194
Question 97 97. You are treating a 5 year-old boy who has recurrent acute otitis media. Which one of the following is the BEST initial management option for this child? A. Tonsillectomy B. PE tube placement C. Adenoidectomy D. Tympanocentesis E. Nasal steroids
195
Answer 97 97. Choice B is the correct answer. This patient needs PE tube placement to allow his middle ear to drain appropriately.
196
Question 98 98. You are evaluating a 23 year-old HIV positive male who complains of severe dysphagia. This is his third bout of dysphagia. Examination reveals oral thrush. Which of the following medications is the MOST appropriate initial therapy? A. Amphotericin B B. Fluconazole C. Griseofulvin D. Nystatin troches E. Penicillin
197
Answer 98 98. Choice B is the correct answer. This patient needs oral fluconazole for esophageal candida.
198
Question 99 99. Which one of the following pathogens is NOT associated with otitis media? A. Varicella-zoster B. Moraxella catarrhalis C. Haemophilus influenzae D. Respiratory syncytial virus E. Streptococcus pneumoniae
199
Answer 99 99. Choice A is the correct answer. Varicella is not associated with otitis media.
200
Question 100 100. Which one of the following in NOT a potential complication of acute mastoiditis? A. Sinusitis B. Meningitis C. Labyrinthitis D. Hearing loss E. Intracranial abscess
201
Answer 100 100. Choice A is the correct answer. Sinusitis is not a potential complication of acute mastoiditis.
202
Question 101 A 12 year-old girl is brought to your clinic by her parents for a complaint of left eye pain. Her eyelid has been red, swollen, hot and painful for the last 2 days. Last night she had a fever of 102ºF. Examination reveals the left upper eyelid is proptosis, edema and erythema extending down into the left cheek. Her visual acuity is decreased on the left. Eyes are PERRLA with pain on EOM movement on the left. Which of the following is the MOST likely diagnosis? Dacryocystitis Mucormycosis Orbital cellulitis Rhabdomyosarcoma Cavernous hemangioma
203
Answer 101 101. Choice C is the correct answer. Orbital cellulitis presents with eyelid redness, fever, proptosis and edema.
204
Question 102 102. A 3 month-old girl has excessive tearing of the right eye since birth and a history of a yellow discharge from the eye. Initially oral antibiotics and massage of the nasal canthus were attempted with minimal to no resolution of symptoms. Examination reveals a normal eye with pooling of tears along the lower lid. Which of the following is the MOST appropriate therapy? Systemic antibiotics Oral steroids Ophthalmology referral for lacrimal stent placement Surgical incision and drainage in the family practice environment Reassurance, it will most likely resolve spontaneously
205
Answer 102 102. Choice E is the correct answer. In this instance reassurance is the best management option that dacryocystitis will resolve spontaneously. If persists can try oral amoxicillin.
206
Question 103 103. A 68 year-old man presents to your clinic with loss of vision in his left eye. He first noted the symptoms while on missionary work in Africa. He notes this is his first opportunity to seek medical care. He noted the abrupt onset of symptoms 3 weeks prior to this visit. Examination reveals an opacified retina with a cherry red spot located proximal to the central fovea. Which of the following is the MOST likely diagnosis? Giant cell arteritis Central retinal vein occlusion Central retinal artery occlusion Retinal arterial macroaneurysm Proliferative diabetic retinopathy
207
Answer 103 103. Choice C is the correct answer. This patient has a central retinal artery occlusion.
208
Question 104 104. Which one of the following conditions is MOST often associated with the wear of new contact lenses or improper cleansing of contact lenses? Viral conjunctivitis Bacterial dacryocystitis Corneal ulceration Giant papillary conjunctivitis Atopic keratoconjunctivitis
209
Answer 104 104. Choice C is the correct answer. Corneal ulceration is associated with the wear of new contact lenses or improper cleansing of contact lenses.
210
Question 105 105. An 18 year-old male reports to your clinic complaining of a sore throat, tearing red eyes, and a fever. He first noted the symptoms approximately 2 days prior to this visit. He states that the symptoms have progressively worsened. Vitals: BP 112/68, P 72, Temp 102.1, R 16. Examination reveals mild scleral injection with lid conjunctival edema, “cobblestoning” of the conjunctival margins, preauricular lymphadenopathy, and clear discharge OU. The pharynx is red with clear drainage. He denies any otalgia or ocular pruritus. Which of the following is the MOST likely diagnosis? Fungal conjunctivitis Inclusion conjunctivitis Blepharoconjunctivitis Hemorrhagic conjunctivitis Viral conjunctivitis
211
Answer 105 105. Choice E is the correct answer. The patient has cobblestoning of his conjunctiva and a viral illness at this time. Viral conjunctivitis is the most likely diagnosis.
212
Question 106 106. While testing a patient for direct and consensual pupillary reaction you note a paradoxical widening of the left pupil when the penlight is shone on it. The pupil constricts normally when the light is shine into the right eye. Which of the following BEST represents with phenomenon? Anisocoria Adie’s pupil Horner’s syndrome Argyll Robertson pupil Afferent Pupillary defect
213
Answer 106 106. Choice E is the correct answer. This patient has an afferent pupillary defect.
214
Question 107 107. Which of the following is a potential early ocular physical finding when evaluating a patient with chronic hypertension on early indication of hypertensive disease? Arteriovenous nicking Papilledema Foveal microaneurysms Circumcorneal thinning Retinal “punch-outs” lesions
215
Answer 107 107. Choice A is the correct answer. AV nicking is a early ocular physical findings seen in patient with chronic hypertension.
216
Question 108 108. A 43 year-old woman is brought to your clinic complaining of sudden onset of blurred vision, pain in the eye, nausea, and vomiting. The symptoms woke her shortly after lying down to take a nap. Examination reveals conjunctival injection. Pupillary response to light is sluggish. There is ciliary injection and corneal cloudiness. Which of the following medications would be indicated for this initial treatment in this patient? Diclofenac Medrysone Dipivefrin Acetazolamide Pilocarpine
217
Answer 108 108. Choice D is the correct answer. Acetazolamide should be given in this instance because this patient has physical findings concerning for glaucoma.
218
Question 109 109. Which of the following is a fundoscopic exam finding commonly associated with proliferative diabetic retinopathy? Drusen bodies Microaneurysms Neovascularization Arteriolar narrowing Retinal hemorrhages
219
Answer 109 109. Choice C is the correct answer. Neovascularization is a physical finding commonly associated with proliferative diabetic retinopathy.
220
Question 110 110. A 73 year-old female with bilateral cataract is asking when she needs to have them removed. Which of the following is the BEST advice for the timing of surgery? When there is functional visual impairment The cataracts obscure the fundoscopic exam The cataracts reach 0.4 mm in size After age 80 When there is a cosmetic concern (the cataract can be visualized by friends and family)
221
Answer 110 110. Choice A is the correct answer. The patient’s cataracts should be removed when there is functional visual impairment.
222
Question 111 111. Which of the following organisms is the MOST likely cause of congenital retinochoroiditis? Cytomegalovirus Toxoplasma gondii Histoplasma capsulatum Herpes simplex virus type 2 Treponema pallidum
223
Answer 111 111. Choice B is the correct answer. Toxoplasma gondii is the most likely organism to cause congenital retinochoroiditis.
224
Question 112 112. A 59 year-old female presents to your clinic with sandy-scratchy sensation of both eyes. Her past medical history is significant for rheumatoid arthritis. Examination reveals the bulbar conjunctiva with thickening and edema. There is fine punctate stippling of the cornea in the interpalpebral fissure. Which of the following topical preparations would be the BEST initial treatment for her condition? Artificial tears Pilocarpine HCL 2% Dexamethasone sodium 0.05% Erythromycin 0.5% Timolol 0.25%
225
Answer 112 112. Choice A is the correct answer. This patient could benefit from artificial tears for dry eye that is common in Rheumatoid Arthritis.
226
Question 113 113. A 33 year-old woman is complaining of transient visual defects and diplopia. Symptoms have been occurring more often over the last six months, often associated with limb paresthesias. Examination reveals a mild optic neuritis and a nystagmus. Which of the following is the MOST likely diagnosis? Pituitary tumor Multiple sclerosis Vitamin B12 deficiency Heavy Metal poisoning Suprasellar Meningioma
227
Answer 113 113. Choice B is the correct answer. MS often presents with diplopia and nystagmus. MS often has optic neuritis associated with it.
228
Question 114 114. Which of the following topical ophthalmic medications is commonly used in the treatment of open angle glaucoma? Rimexolone Cromolyn sodium Pilocarpine Scopolamine Trifluridine
229
Answer 114 114. Choice C is the correct answer. Pilocarpine is used in the treatment of open angle glaucoma.
230
Question 115 115. A 34 year-old female reports to your clinic complaining of bilateral eye irritation and injection with a purulent discharge that is worse in the morning and sticky with a little lid edema. Further interview reveals that she started doing volunteer work at child day care center four weeks prior to this visit. Which of the following organisms is the MOST likely cause? Escherichia Coli Coccidioides immitis Neisseria gonorrhoeae Chlamydia trachomatis Streptococcus pneumoniae
231
Answer 115 115. Choice E is the correct answer. Streptococcus pneumoniae is the most common organism causing bacterial conjunctivitis.
232
Question 116 116. A 36 year-old truck driver has had mild left eye pain that seems to be associated with a “mass” in his eye. Examination reveals yellow nodule on the nasal aspect, near the palpebral aperture. There is moderate level of vascular injection and no discharge. Remainder of the examination is negative. Which of the following is the MOST likely diagnosis? Pterygium Cystinosis Pinguecula Droplet keratopathy Congenital conjunctival lymphoma
233
Answer 116 116. Choice C is the correct answer. This patient has a Pinguecula.
234
Question 117 117. Which of the following topical ophthalmic agents is indicated for prophylaxis of ophthalmia neonatorum due to N gonorrhoeae or C trachomatis? Ofloxacin 0.3% Sulfacetamide sodium 10% Erythromycin 0.5% Prednisolone acetate 1% Nedocromil sodium 2%
235
Answer 117 117. Choice C is the correct answer. Erythromycin is the best agent for prophylaxis of ophthalmia neonatorum due to N. gonorrhoeae or C. trachomatis.
236
Question 118 118. If left untreated, congenital syphilis could commonly produces what ocular manifestation? Amblyopia Inflammatory scleritis Occlusion dacryocystitis Interstitial keratitis Anterior uveitis
237
Answer 118 118. Choice D is the correct answer. If left untreated interstitial keratitis will result from congenital syphilis.
238
Question 119 A 28 year-old male HIV patient reports to your clinic for a routine follow-up. He denies any symptoms and states that he has had good compliance with his therapy. There are yellowish-white patches of retinal opacification and retinal hemorrhages noted on dilated fundoscopic exam. CD4 count is 48/μL. Which of the following organisms is the MOST COMMON cause of these findings? Cytomegalovirus Toxoplasma gondii Histoplasma capsulatum Herpes simplex virus type 2 Treponema pallidum
239
Answer 119 119. Choice A is the correct answer. Cytomegalovirus is the most common cause of Retinitis is an HIV patient with a low CD4 count.
240
Question 120 120. A 28 year-old male HIV patient reports to your clinic for a routine follow-up. He denies any symptoms and states that he has had good compliance with his therapy. There are yellowish-white patches of retinal opacification and retinal hemorrhages noted on dilated fundoscopic exam. CD4 count is 48/μL. Treatment for this patient should include: Topical fluorometholone 0.1% Topical bacitracin 500 units/gm Intravitreal ganciclovir 4.5 mg Oral dexamethasone 20 mg Oral acyclovir 400 mg
241
Answer 120 120. Choice C is the correct answer. Intravitreal ganciclovir is the treatment of choice of CMV retinitis.
242
Question 121 121. The visual loss associated with optic neuritis often presents as a/an: Central scotoma. Bilateral temporal deficit. Sudden loss in the superior fields. Painless complete loss upon waking. Midline "blurriness" that resolves when the patient rubs their eyes.
243
Answer 121 121. Choice A is the correct answer. The visual loss associated with optic neuritis presents as a central scotoma.
244
Question 122 122. Papilledema is commonly caused by:
Blunt trauma to the eye. Bacterial conjunctival infection. Increased intracranial pressure. Untreated open angle glaucoma. Prolonged contact lens wear.
245
Answer 122 122. Choice C is the correct answer. Papilledema is seen with increased intracranial pressure. It is a common finding in patients with pseudotumor cerebri.
246
Question 123 123. Which of the following conditions is NOT considered a cause of Amaurosis fugax? Sickle cell anemia Cardiac dysrhythmias Carotid artery stenosis Diabetes mellitus Valvular heart disease
247
Answer 123 123. Choice D is the correct answer. Diabetes mellitus is not considered a cause of Amaurosis Fugax.
248
Question 124 124. Which of the following conditions is MOST commonly associated with the presence of typical Argyll Robertson pupils? Diabetes Encephalitis Parkinsonism Chronic Alcoholism Central nervous system syphilis
249
Answer 124 124. Choice E is the correct answer. The most common condition associated with a typical Argyll Robertson Pupil is central nervous system syphilis.
250
Question 125 125. Which of the following conditions is BEST characterized by the clinical triad of keratoconjunctivitis sicca, xerostomia, and connective tissue disease? Still’s disease Ulcerative colitis Dermatomyositis Sjogren’s syndrome Wegener’s granulomatosis
251
Answer 125 125. Choice D is the correct answer. Sjogren’s syndrome is characterized by the triad of keratoconjunctivitis, sicca and xerostomia.
252
Question 126 126. A 24-year-old nursing student in for occupational physical reports that she is “near sighted”. Which of the following descriptions BEST reflects the definition of this refractive error? The image of distant objects focuses in front of the retina. The eye produces an image with multiple focal points. The image of objects focuses behind the retina. There is the loss of accommodation. There is a refractive difference between the horizontal and vertical axes.
253
Answer 126 126. Choice A is the correct answer. Near sighted means that the image of distant objects focuses in front of the retina.
254
Question 127 127. A 15 year-old male reports to your clinic complaining of "itchy, red eyes" intermittently over the last five weeks. He also notes that he rubs his eyes a lot especially after playing football. Examination reveals a sunken eye appearance, increased lacrimation and clear discharge that is scant and ropy. Which of the following is the MOST LIKELY diagnosis? Atopic keratoconjunctivitis Cicatricial pemphigoid Allergic conjunctivitis Reiter’s syndrome Phlyctenulosis
255
Answer 127 127. Answer C is the correct answer. This patient has allergic conjunctivitis.
256
Question 128 128. A 15 year-old male reports to your clinic complaining of "itchy, red eyes" intermittently over the last five weeks. He also notes that he rubs his eyes a lot especially after playing football. Examination reveals a sunken eye appearance, increased lacrimation and clear discharge that is scant and ropy. Which of the following topical ophthalmic agents are indicated in this patient? Levocabastine HCl Chloramphenicol Erythromycin Medrysone Natamycin
257
Answer 128 128. Choice A is the correct answer. Levocabastine is indicated for this patient with allergic conjunctivitis.
258
Question 129 129. An 80 year-old woman presents to your clinic with a three-year history of eyelid irritation. She complains that both eyelids burn and itch all the time. Examination reveals mild lid margin erythema and scaling with a greasy consistency. Which of the following is the BEST initial treatment? Gently wash the lid margins with baby shampoo Application of petroleum jelly at lid margins Topical prednisolone 0.125% Oral amikacin 250 mg Topical trifluridine 1%
259
Answer 129 129. Choice A is the correct answer. Gently washing the lid margins with baby shampoo is the best initial treatment for blepharitis.
260
Question 130 130. A 23 year-old male reports to your clinic complaining of a "foreign body sensation" when he blinks his right eye. Examination reveals a small lesion at the upper lid margin, near the midline. The lesion is swollen, red and tender and is pointing toward the conjunctiva. Which of the following is the MOST likely diagnosis? Chalazion Coloboma Blepharitis Hordeolum Dacryocystitis
261
Answer 130 130. Choice D is the correct answer. This patient has a hordeolum.
262
Question 131 131. Which of the following tests is has the highest diagnostic yield when evaluating a patient with a suspected corneal abrasion? Inspection of the cornea with tangential lighting Indirect ophthalmoscopy Fluorescein staining Lid eversion Tonometry
263
Answer 131 131. Choice C is the correct answer. Fluorescein staining has the highest yield in patients with suspected corneal abrasion.
264
Question 132 132. In evaluating the “red eye”, which of the following conditions would MOST LIKELY be associated with abnormalities in pupillary size? Keratitis Dacryocystitis Thyrotoxicosis Bacterial conjunctivitis Acute angle-closure glaucoma
265
Answer 132 132. Choice E is the correct answer. Acute angle closure glaucoma would have a red eye with possibly having some abnormalities of pupils.
266
Question 133 133. A 24 year-old female patient presents to your clinic complaining of severe pain in both of her eyes. She states that yesterday she welded together of sculpture for an art exhibit at a local college. Fluorescein stain reveals a corneal defect. Which of the following is the BEST initial treatment? Binocular patching and instillation of cyclopentolate Binocular patching and installation of fluorometholone Patch the affected eye and oral ciprofloxacin Patch the affected eye and oral gentamicin No patch and cool compresses only
267
Answer 133 133. Choice E is the correct answer. The patient should not have her eyes patched. She should apply cool compresses for welders burns.
268
Question 134 134. Which of the following ocular topical medications could potentially exaggerate intraocular pressure and subsequently simulate a primary open-angle glaucoma? Polymyxin B Sulfate Dexamethasone sodium Nedocromil sodium Metipranolol HCl Apraclonidine HCl
269
Answer 134 134. Choice B is the correct answer. Dexamethasone could possibly exaggerate intraocular pressure and stimulate primarily open angle glaucoma.
270
Question 135 135. Paralysis of which cranial nerve causes a convergent squint in the primary position with failure of abduction of the affected eye, producing horizontal diplopia that increases on gaze to the affected side and on looking into the distance? Second Third Fourth Fifth Sixth
271
Answer 135 135. Choice E is the correct answer. Paralysis of the sixth cranial nerve causes a convergent squint in the primary position with failure of abduction of affected eye.
272
Question 136 136. Paralysis of which cranial nerve causes upward deviation of the eye with failure of depression on adduction? Second Third Fourth Fifth Sixth
273
Answer 136 136. Choice C is the correct answer. Paralysis of the Fourth CN causes upperward deviation and failure of depression on adduction.
274
Question 137 137. A 31 year-old woman reports to your clinic complaining of moderate light sensitivity and moderate to severe right eye pain for 3 days. She states that her eye is constantly watering and "gets redder by the day". She also notes that her vision is slightly blurred. Visual acuity uncorrected OD 20/80 (usually 20/20 by history) OS 20/20. Examination reveals the right eye is moderately injected with a miotic pupil that is photophobic to direct and consensual penlight challenge. Slit lamp examination reveals fine white deposits on the posterior surface of the cornea. Which of the following is the MOST likely diagnosis? Anterior uveitis Posterior uveitis Scleritis Episcleritis Angle closure glaucoma
275
Answer 137 137. Choice A is the correct answer. This patient has anterior uveitis.
276
Question 138 138. A 32-year-man has a unilateral miosis, ptosis, and absence of sweating on the ipsilateral face and neck. Which of the following is the MOST likely diagnosis? CN XI palsy Bell’s palsy Adie’s syndrome Horner’s syndrome Myasthenia gravis
277
Answer 138 138. Choice D is the correct answer. Horner’s syndrome causes unilateral miosis, ptosis, and absence of sweating on the ipsilateral face and neck.
278
Question 139 139. A 68 year-old female presents to your clinic with a chief complaint of severe left side temporal headache and pain with chewing. She also notes a loss of vision that "comes and goes" particularly in the lower one-half or inferior visual field of the involved side. Family history and past medical history provide no important information. Vitals: BP 150/90, P 88, Temp 99.2, R 18. DVA OS (uncorrected) 20/80 OD (uncorrected) 20/30. PE reveals marked tenderness over the left temporal region and a pale, swollen optic disc with splinter hemorrhages. Initial Sedimentation Rate (ESR) is 68 (0 to 20 = normal). All other labs are pending. Treatment for this patient should include: IV antibiotics IV steroids PO Nonsteroidal anti-inflammatory medications OS topical antibiotics Withhold all therapy until a confirmed diagnosis is made
279
Answer 139 139. Choice B is the correct answer. This patient treatment should include IV steroids.
280
Question 140 140. A 27 year-old female reports to your clinic complaining of severe right eye pain and a "really red eye". She states the symptoms have been progressively worsening for the last two weeks, "I thought I was just having problems with my contacts." She states that she occasionally sleeps with her corrective soft contact lenses in and she has occasionally cleans them with tap water when she runs out of her cleaning agents. She denies all other all other symptoms and she has no other past medical history. Vitals: BP 112/68, P 68, R 14, Temp 98.2, DVA (uncorrected) OD 20/100 OS 20/200. On physical you note moderate conjunctival injection and thick mucopurulent discharge. On slit-lamp exam you note perineural and ring infiltrates in the corneal stroma. Which of the following is the MOST likely diagnosis? Infectious Mucormycosis Acanthamoeba Keratitis Viral dacryocystitis Posterior uveitis Amebic episcleritis
281
Answer 140 140. Choice B is the correct answer. This patient has Acanthamoeba Keratitis.
282
Question 141 141. A 10 year-old male is brought into your clinic for "suddenly being afraid of the dark." The patient insists he’s not afraid, "I just don't see things good, especially at night". His visual acuity is normal. Funduscopic examination reveals narrowing of the arterioles and a mottled appearance of the retina. Which of the following is the MOST likely diagnosis? Spasmus nutans Retinitis pigmentosa Macular degeneration Retrolental fibroplasia Cystoid macular edema
283
Answer 141 141. Choice B is the correct answer. Retinitis pigmentosa is an inherited progressive degeneration of the retina, affecting photoreceptor and pigment epithelial dysfunction.
284
Question 142 142. In which of the following patients would have the HIGHEST potential of developing fungal keratitis? 28 year-old male immigrant farmer 30 year-old male with history of asthma 32 year-old female pregnant with gestational diabetes 38 year-old female two pack per day smoker 48 year-old male with uncontrolled hypertension and hyperlipidemia
285
Answer 142 142. Choice A is the correct answer. The 28 year old immigrant farmer would have the highest potential of fungal keratitis.
286
Question 143 A 79 year-old male reports to your clinic complaining of headache, dizziness, tinnitus, blurred vision, and fatigue. He also notes generalized pruritus that worsens following a warm shower. Funduscopic examination reveals engorgement of the retinal arteries and veins. There are hemorrhages, microaneurysms and areas of capillary closure present. Which of the following conditions is MOST likely to produce these findings? Leukemia Polycythemia vera Multiple sclerosis Sickle cell disease Metastatic melanoma from a distant site
287
Answer 143 143. Choice B is the correct answer. Polycythemia vera would be most likely to cause these findings on physical exam.
288
Question 144 144. Which of the following statements about presbyopia is TRUE? Typically seen in patients between the ages of 30 and 35. Rapidly changes far vision following onset. Easily correctable with surgery. Caused by the natural loss of visual accommodative capacity. Confrontation testing is the gold standard for diagnosis.
289
Answer 144 144. Choice D is the correct answer. Presbyopia is caused by the natural loss of visual accommodative capacity.
290
Question 145 145. Which of the following types of cataracts are the most common? Congenital Metabolic Traumatic Senile Systemic
291
Answer 145 145. Choice D is the correct answer. Senile cataracts are the most common cataracts.
292
Question 146 146. Which of the following conditions or activities is associated with age-related macular degeneration? Contact sports Alcohol consumption Cigarette smoking Obesity High protein/low carbohydrate diet
293
Answer 146 146. Choice C is the correct answer. Cigarette smoking is associated with age related macular degeneration.
294
Question 147 147. Which of the following ocular conditions has the potential of developing in a patient with central retinal vein occlusion? Neovascular glaucoma Chronic episcleritis Unilateral cataract Autoimmune corneal ulcer Keratoconjunctivitis sicca
295
Answer 147 Choice A is the correct answer. Neovascular glaucoma is potential consequence of central retinal vein occlusion.
296
Question 148 148. You are evaluating a 48-year-old male for possible primary open-angle glaucoma. When considering the fundoscopic exam, which one of the following would have the highest diagnostic value? A:V ratio Cup:disc ratio Macular exam Lens clarity Vitreous clarity
297
Answer 148 148. Choice B is the correct answer. The cup:disc ratio has the highest diagnostic value in evaluating patients with primary open angle glaucoma.
298
Question 149 149. Which of the following ocular medications could potentially precipitate hypertensive crisis and angina? Timolol Pilocarpine Dorzolamide Dipivefrin Phenylephrine
299
Answer 149 149. Choice E is the correct answer. Phenylephrine because it is an alpha blocker could precipitate hypertensive crisis and angina.
300
Question 150 150. Which of the following oral medications could potentially cause disturbances of color vision, scotomas, and photopsia? Quinidine Haloperidol Vigabatrin Digitalis Chloramphenicol
301
Answer 150 150. Choice D is the correct answer. Digitalis could cause disturbance of colors in vision, scotomas, or photopsia.
302
Question 151 151. Which of the following sinuses is LAST to be fully developed in an infant and may not appear as air-filled spaces until 10 years of age? Frontal Mastoid Ethmoid Maxillary Sphenoid
303
Answer 151 151. Choice A is the correct answer. The frontal sinuses is the last to be fully developed and may not appear as air filled spaces until the age of 10.
304
Question 152 152. A mother is called to pick up her child from daycare after her 4 year-old girl had a mucopurulent discharge from her right eye throughout the day. On exam, she appears to be mildly photophobic but has normal (20/25) vision. Inspection reveals conjunctival injection and edema, with a “cobblestone” appearance. Which of the following is the MOST likely diagnosis? Blepharitis B. Dacryocystitis C. Orbital cellulitis D. Posterior uveitis E. Conjunctivitis
305
Answer 152 152. Choice E is the correct answer. This patient has conjunctivitis.
306
Question 153 153. A 3 year-old boy with a history of recurrent otitis media has increased ear pain for the last 24 hours. The child is inconsolable and has profound anorexia and mild diarrhea. On examination, he is febrile and has right post-auricular tenderness and erythema and the pinna is displaced downward and outward. Which of the following is the MOST likely diagnosis? A. Foreign body inflammation B. Malignant otitis externa C. Mononucleosis D. Otitis media E. Mastoiditis
307
Answer 153 153. Choice E is the correct answer. This patient has mastoiditis.
308
Question 154 154. A 25-year-old male presents to the emergency department with uniocular conjunctivitis. Examination of the eye reveals a dendriform corneal ulcer (without infiltrate), which is well demonstrated with fluorescein staining. Further treatment and consultation is for: Epidemic keratoconjunctivitis Herpes simplex keratoconjunctivitis Chlamydial conjunctivitis Herpes zoster ophthalmicus Congenital ulcerative disorder
309
Answer 154 154. Choice B is the correct answer. Ulcers with dendriform characteristics are consistent with herpetic infections.
310
Question 155 155. A 16-year-old male presents to the Emergency Department with a complaint of a red, hot, and swollen right eyelid. On exam, you find that the bulbar conjunctiva and cornea are clear, with acuity and pupillary reaction maintained and full ocular motility preserved. The correct treatment is: oral antibiotics IV antibiotics Hospital admission CT scan Systemic work-up
311
Answer 155 155. Choice A is the correct answer. This patient has full ocular motility and normal visual acuity. This patient can attempt to be treated as an outpatient with oral antibiotics with close follow up.
312
Question 156 156. Which of the following techniques for removing nasal foreign bodies is recommended for an uncooperative pediatric patient without the use of restraints or conscious sedation? Suction catheter Alligator forceps Magill forceps Fogarty catheter Positive pressure
313
Answer 156 Choice E is the correct answer. Positive pressure does not require restraints. This is done by apply positive pressure to the mouth and occluding the contralateral nare.
314
Question 157 157. You are performing a slit lamp examination on a 30-year-old male complaining of left eye pain. You notice the appearance of "headlights in a fog" as you are evaluating the anterior chamber of that eye. What does this finding indicate? Inflammation Normal finding Blood in the aqueous Layering of white cells Pupillary paralysis
315
Answer 157 157. Choice A is the correct answer. Headlights in a fog appearance in the anterior chamber is seen with inflammation such as in iritis.
316
Question 158 158. You are operating in a field environment when a 43-year-old male patient presents with a posterior nosebleed. A commercial device for posterior nasal packing is not available so you decide to use a Foley catheter. After trimming the tip of the Foley catheter and inserting it through the nose until the distal balloon tip is visualized in the posterior pharynx, the balloon is inflated with _______________ before pulling it anteriorly. 4-8 ml of water 2 ml of gel foam 20 ml of air 15 ml of normal saline 10 ml of lidocaine
317
Answer 158 158. Choice A is the correct answer. There should be 4-8 mL injected of water injected in the balloon before pulling it anteriorly.
318
Question 159 159. A 1 year-old patient weighing 10 kg presents with a fever of 103 F along with left ear pain, nasal congestion and decreased appetite. On exam, the right tympanic membrane is erythematous and bulging. The correct dosage of Amoxicillin based on weight (assuming the dosage is 90mg /kg/day and that you are using the 250mg/5ml suspension divided TID) is: 5 ml TID 7 ml TID 7.5 ml TID 8 ml TID 10 ml TID
319
Answer 159 159. Choice B is the correct answer. The correct dosage for the weight is 7 mL PO TID.
320
Question 160 160. A 3 year-old presents to the clinic with new onset of a “lazy eye” his mother has noted intermittently over the past few months. In particular, she notices his eye deviates inward when looking at things close up and is worse when he gets tired. A cover/uncover test is performed showing some medial deviation of the left eye. The MOST APPROPRIATE management of this patient is: Reassurance since the esotropia is intermittent Reassurance since at this age the patient will most likely outgrow the esotropia. Follow-up in 6 months to monitor for progression. CT scan to look for brain tumor. Referral to ophthalmology.
321
Answer 160 160. Choice E is the correct answer. This patient should be referred to ophthalmology for strabismus.
322
Question 161 161. Which of the following statements regarding acute bacterial rhinosinusitis in children is TRUE? The maxillary and ethmoidal sinuses are the most commonly involved sinuses. Acute sinusitis is diagnosed if postnasal drip and cough persist longer than 3 days. Negative transillumination of the sinuses confirms the diagnosis. A plain x-ray showing air-fluid levels is indicated to confirm the diagnosis. A CT scan showing mucosal thickening is indicated to confirm the diagnosis.
323
Answer 161 161. Choice A is the correct answer. The maxillary and ethmoid sinuses are the most commonly involved sinuses with sinusitis.
324
Question 162 162. A 10 year-old presents with a 1 day history of a red left eye with discharge. He states that the discharge is whitish and caused his eyelid to “stick shut” in the morning. He denies photophobia or vision loss but admits that it feels like he has sand in his eye when he blinks. He denies any symptoms in his right eye but does have some nasal discharge. On exam he has matted lashes and a mucoid discharge. Vision is 20/20. The conjunctiva is red and swollen with and left preauricular lymphadenopathy is noted. The MOST LIKELY diagnosis is: Bacterial conjunctivitis Allergic conjunctivitis Viral conjunctivitis Kawasaki disease Dacryocystitis
325
Answer 162 162. Choice C is the correct answer. This patient likely has viral conjunctivitis given his URI symptoms, sand feeling in his eye, and some whitish discharge that caused his eye to stick shut in the morning.
326
Question 163 163. What is the mechanism of action for Cromolyn sodium in the treatment of allergic rhinitis? Bronchodilator Vasoconstriction Secretion Stimulation Mast cell stabilizer Anti-inflammatory
327
Answer 163 163. Choice D is the correct answer. Cromolyn sodium is a mast cell stabilizer.
328
Question 164 164. A 16 year-old boy presents with one week of fatigue, anorexia, sore throat and fevers up to 102 F. His father reports that he has been sleeping 2-3 hours more than usual per day. Exam shows an ill-appearing boy with a red throat, moderate exudate, cervical and inguinal lymphadenopathy and mild splenomegaly. Which virus is the MOST LIKELY pathogen? Poliovirus Epstein-Barr virus Varicella zoster Herpes simplex Parvovirus B19
329
Answer 164 164. Choice B is the correct answer. This patient is likely infected with Epstein Barr virus that causes mononucleosis. This causes the fever, splenomegaly, cervical and inguinal lymphadenopathy, as well as the exudative pharyngitis.
330
Question 165 165. A school-age boy suffered from a sore throat three weeks ago. Now he complains of fever, fatigue, and pain in both knee joints. The involved joints are tender, hot and swollen. Which of the following is the MOST LIKELY diagnosis? Septic arthritis Rheumatic fever Juvenile rheumatoid arthritis Sickle cell crisis Erythema nodosum
331
Answer 165 165. Choice B is the correct answer. This patient likely has rheumatic fever secondary to an untreated strep infection 3 weeks prior to the onset of arthralgia, fatigue, and fever.
332
Question 166 166. A 4-year-old boy is brought to the Emergency Department by his mother who says, “He’s got something stuck in his ear”. Of the following, what is TRUE concerning removal of foreign bodies from the ear? Irrigation can be used if the tympanic membrane is partially visualized . Irrigation is recommended for removing organic materials In children, foreign bodies medial to the bony isthmus can be removed safely without sedation Live objects should be drowned with 2% lidocaine solution or viscous lidocaine before removal Antibiotics should be prescribed for all patients who undergo foreign body removal
333
Answer 166 166. Choice D is the correct answer. Live objects should be drowned in 2% lidocaine prior to removal.
334
Question 167 167. A 75-year-old male comes to the Emergency Department complaining of a visual loss in his right eye. He states that the vision “went out” suddenly about 20 minutes ago and that there was no pain associated with it. Past medical history is significant for long-standing hypertension. Fundoscopic exam reveals optic disc edema and diffuse retinal hemorrhages in all quadrants. The contralateral optic nerve and fundus are normal What is the diagnosis? Acute optic neuritis . Acute vitreous hemorrhage Acute angle closure glaucoma Central retinal vein occlusion Central retinal artery occlusion
335
Answer 167 167. Choice D is the correct answer. This patient has history and physical findings associated with central retinal vein occlusion.
336
Question 168 168. A 62-year-old diabetic presents to the ED with left ear and left sided facial pain. He states that his primary provider has been treating him for an outer ear infection for the past 2 weeks and that it is “just getting worse”. Vital signs are: BP-140/88, P-90, T F, R-18. On exam, the patient has extreme pain with manipulation of the auricle, erythema and edema of the EAC, and parotitis. The tympanic membrane cannot be visualized due to the edema. What should you suspect in this patient? Acute diffuse otitis externa Bullous myringitis Malignant otitis externa Acoustic neuroma Periauricular abscess
337
Answer 168 168. Choice C is the correct answer. This patient has malignant otitis externa demonstrated by pain with manipulation of auricle, edema and erythema of the EAC with parotitis.
338
Question 169 169. A 15-year-old female got hit in her right eye with a softball and now presents with an obvious hyphema. What manual procedure can you perform in a prehospital or hospital setting prior to the use of medications? Gentle massage to prevent clogging of suspended RBCs Eye shielding to promote “pupillary play” Direct pressure to decrease intraocular pressure Head elevation to settle suspended RBCs inferiorly Indirect lighting to prevent iris root vessel stretch
339
Answer 169 Answer D is correct answer. Head elevation to settle suspend RBC’s inferiorly.
340
Question 170 170. You are evaluating a 19 year old female for right ear pain. She states she has been taking Motrin four times a day for the pain without much improvement. Which one of the following is the MOST likely cause of this otalgia? Cholesteatoma Otosclerosis Presbycusis Ototoxicity Bruxism
341
Answer 170 170. Choice E is the correct answer. Bruxism is a condition where you grind your teeth and this can cause ear pain because of TMJ strain. The rest of these conditions are essentially painless conditions.
342
Question 171 171. You are evaluating a patient who complains of decreased hearing in the left ear for about one week. The patient denies tinnitus and vertigo. Your Weber test lateralizes to the left. You Rinne test shows BC>AC in the left ear. Which one of the following is NOT a cause of this type hearing loss? Ossicular disruption Acoustic neuroma Otosclerosis Otitis media with effusion External auditory canal obstruction
343
Answer 171 171. Choice B is the correct answer. Acoustic neuroma would not cause this type of hearing loss.
344
Question 172 172. During the evaluation of vertigo it is important to differentiate peripheral causes from central causes. Which one of the following is LEAST likely to be associated with central vertigo? Cranial nerve palsies. Motor deficits. Sensory deficits. Normal auditory function. Tinnitus
345
Answer 172 172. Choice E is the correct answer. Tinnitus is the least likely to cause central vertigo.
346
Question 173 173. You are evaluating a 44 year-old male who complains of nasal congestion, pressure around his eyes and forehead, runny nose, and scratchy throat. Physical exam reveals; BP 150/86, P 68, R 12, T 101.1, TMs retracted bilaterally, edematous nasal mucosa, clear rhinorrhea, and scant post nasal drainage. Which one of the following is the MOST COMMON complication of this condition? Chronic Sinusitis Middle ear effusion Mastoiditis Nasal vestibulitis Cholesteatoma
347
Answer 173 173. Choice B is the correct answer. The most common complication of this condition is a middle ear effusion.
348
Question 174 174. A 22 year-old female patient presents to sick-call with 12-day history of cough, runny nose with congestion and a sore throat. This AM she awoke with a pounding "facial headache" and what she perceives as swelling around her eyes. She states that her headache is made worse by bending forward to tie her shoes. VS: T: 100.6o F, R: 20, P: 92, BP: 129/65. EXAM: Bilateral TM's clear with decreased mobility, nares bilateral swollen mucosa with purulent rhinorrhea, right greater than left maxillary sinus tenderness to percussion, oral cavity is erythemic without exudate, neck supple with lymphadenopathy, chest clear to auscultation bilaterally. Which one of the following is the most likely diagnosis? Allergic rhinitis Viral upper respiratory infection Strep pharyngitis Right maxillary sinusitis Viral bronchitis
349
Answer 174 174. Choice D is the correct answer. This patient likely has right maxillary sinusitis.
350
Question 175 175. A 25 year-old male patient presents with a primary complaint of recurrent epistaxis over the past 5 days. The patient has not been able to control the bleeding despite repeated application of pressure to his nose. The patient has also noted some blood on his toothbrush after brushing which he says is unusual for him. The patient denies taking any medicines recently and specifically denies the use of aspirin. VS are notable for a fever to degrees F and tachycardia (pulse 110). Physical exam is notable for scattered petechiae on his oral mucosa and prominent gingival hypertrophy. There is no noted adenopathy. Initial laboratory work includes a CBC which shows leukocytosis (WBC 125,000) and thrombocytopenia (platelet count 40,000). What is the MOST likely diagnosis? Multiple myeloma Hodgkin’s Disease Acute leukemia Idiopathic thrombocytopenic purpura (ITP) Surreptitious use of aspirin
351
Answer 175 175. Choice C is the correct answer. This patient likely has acute leukemia.
352
Question 176 176. All of the following infectious agents are well known causes of primary acute tonsillitis EXCEPT staphylococci streptococci viral agent diphtheria organisms Neisseria gonorrhoeae
353
Answer 176 176. Choice A is the correct answer. Staphylococci is not a well known cause of acute tonsillitis.
354
Question 177 177. The most common presenting symptoms of acute sinusitis in children are: Headache, facial pain and fever Protracted nasal discharge and cough Poor olfactory sense Cough and wheezing at night Nasal voice and congestion
355
Answer 177 177. Choice B is the correct answer. Protracted nasal discharge and cough are the most common presenting symptoms of acute sinusitis.
356
Question 178 178. You are seeing a 32-year-old patient for follow up for his sore throat that started 5 days ago. He was started on 500mg Pen VK po QID yesterday for presumed Strep pharyngitis. He now complains of a more severe pain on the left side of his throat and it hurts to open his mouth. On exam you note slight deviation of the soft palate and uvula to the right. Which one of the following should you do for this patient? Switch the patient to Augmentin to cover resistant strains of Strep Do not change treatment plan because there has not been enough time for the antibiotic to take effect Prescribe narcotic analgesics for the pain Check a Mono-spot because the patient most likely has mononucleosis Aspirate the left peritonsillar fold to evaluate for an abscess
357
Answer 178 178. Choice E is the correct answer. The left peritonsillar fold should be aspirated to evaluate for abscess by ENT.
358
Question 179 179. What is the MOST COMMON cause of pharyngitis?
Haemophilus influenzae Gram negative intracellular diplococci Viral upper respiratory tract pathogens Group A Beta hemolytic Streptococcus Infectious mononucleosis
359
Answer 179 179. Choice C is the correct answer. Viral upper respiratory tract pathogens are the most common cause of pharyngitis.
360
Question 180 180. You are evaluating a 28-year-old male who complains of a tender mass on the left side of his neck. He first noticed it five days ago. The mass has gotten larger over the last few days. The patient denies any additional symptoms. On exam you note a slightly tender, non-mobile mass at the anterior aspect of the sternocleidomastoid muscle. The mass does not move when you have the patient swallow. Which one of the following is the MOST LIKELY diagnosis? Mycobacterial lymphadenitis Hodgkin’s Lymphoma Inflamed brachial cleft cyst Thyroglossal duct cyst Reactive lymphadenopathy
361
Answer 180 180. Choice C is the correct answer. This is likely an inflamed brachial cleft cyst.
362
Question 181 181. You are evaluating a 22 year-old male in the ER who was involved in a bar fight and was punched in the nose about one hour ago. There is copious blood on his T-shirt, his nose is noticeably deformed and you note a widened, purplish nasal septum. If left untreated, which one of the following is a likely complication? The nasal septum will become infected The nasal bone deformity will be irreversible The patient will develop a saddle nose deformity The patient will develop anosmia The patient will develop nasal polyps
363
Answer 181 181. Choice C is the correct answer. This patient has a septal hematoma and will develop a saddle nose deformity.
Similar presentations
© 2025 SlidePlayer.com. Inc.
All rights reserved.